Đến nội dung

Bachhh

Bachhh

Đăng ký: 17-12-2016
Offline Đăng nhập: 01-04-2017 - 22:35
-----

#672459 $\frac{a+b+c}{3}\geq \frac{1}{4}\sqrt[3]{\frac{...

Gửi bởi Bachhh trong 23-02-2017 - 07:05

Cho a,b,c >0 CMR:

Hình gửi kèm

  • MathMagic170223_1.png